Difference between revisions of "2000 AMC 10 Problems/Problem 17"

m (Problem)
 
(4 intermediate revisions by 4 users not shown)
Line 1: Line 1:
 
==Problem==
 
==Problem==
  
Boris has an incredible coin changing machine. When he puts in a quarter, it returns five nickels; when he puts in a nickel, it returns five pennies; and when he puts in a penny, it returns five quarters. Boris starts with just one penny. Which of the following amounts could Boris have after using the machine repeatedly?
+
Boris has an incredible coin-changing machine. When he puts in a quarter, it returns five nickels; when he puts in a nickel, it returns five pennies; and when he puts in a penny, it returns five quarters. Boris starts with just one penny. Which of the following amounts could Boris have after using the machine repeatedly?
  
<math>\mathrm{(A)}</math> <math>\$3.63</math>
+
<math> \textbf{(A)} \$3.63 \qquad \textbf{(B)} \$5.13 \qquad \textbf{(C)}\$6.30 \qquad \textbf{(D)} \$7.45 \qquad \textbf{(E)} \$9.07</math>
 
 
<math>\mathrm{(B)}</math> <math>\$5.13</math>
 
 
 
<math>\mathrm{(C)}</math> <math>\$6.30</math>
 
 
 
<math>\mathrm{(D)}</math> <math>\$7.45</math>
 
 
 
<math>\mathrm{(E)}</math> <math>\$9.07</math>
 
  
 
==Solution==
 
==Solution==
Line 19: Line 11:
 
This implies that the only possible values, in cents, he can have are the ones one more than a multiple of <math>124</math>. Of the choices given, the only one is <math>\boxed{\text{D}}</math>
 
This implies that the only possible values, in cents, he can have are the ones one more than a multiple of <math>124</math>. Of the choices given, the only one is <math>\boxed{\text{D}}</math>
  
==Video Solution by WhyMath==
+
==Video Solution==
 
https://youtu.be/ZmOrAsgvS4s
 
https://youtu.be/ZmOrAsgvS4s
  
 
~savannahsolver
 
~savannahsolver
 +
 +
https://youtu.be/oWxqYyW926I
 +
 +
-gnv12
  
 
==See Also==
 
==See Also==
Line 28: Line 24:
 
{{AMC10 box|year=2000|num-b=16|num-a=18}}
 
{{AMC10 box|year=2000|num-b=16|num-a=18}}
 
{{MAA Notice}}
 
{{MAA Notice}}
 +
[[Category:Introductory Number Theory Problems]]

Latest revision as of 03:52, 20 July 2023

Problem

Boris has an incredible coin-changing machine. When he puts in a quarter, it returns five nickels; when he puts in a nickel, it returns five pennies; and when he puts in a penny, it returns five quarters. Boris starts with just one penny. Which of the following amounts could Boris have after using the machine repeatedly?

$\textbf{(A)} $3.63 \qquad \textbf{(B)} $5.13 \qquad \textbf{(C)}$6.30 \qquad \textbf{(D)} $7.45 \qquad \textbf{(E)}  $9.07$

Solution

Consider what happens each time he puts a coin in. If he puts in a quarter, he gets five nickels back, so the amount of money he has doesn't change. Similarly, if he puts a nickel in the machine, he gets five pennies back and the money value doesn't change. However, if he puts a penny in, he gets five quarters back, increasing the amount of money he has by $124$ cents.

This implies that the only possible values, in cents, he can have are the ones one more than a multiple of $124$. Of the choices given, the only one is $\boxed{\text{D}}$

Video Solution

https://youtu.be/ZmOrAsgvS4s

~savannahsolver

https://youtu.be/oWxqYyW926I

-gnv12

See Also

2000 AMC 10 (ProblemsAnswer KeyResources)
Preceded by
Problem 16
Followed by
Problem 18
1 2 3 4 5 6 7 8 9 10 11 12 13 14 15 16 17 18 19 20 21 22 23 24 25
All AMC 10 Problems and Solutions

The problems on this page are copyrighted by the Mathematical Association of America's American Mathematics Competitions. AMC logo.png